Đến nội dung

Khoa Linh nội dung

Có 599 mục bởi Khoa Linh (Tìm giới hạn từ 27-04-2020)



Sắp theo                Sắp xếp  

#701385 $\boxed{\text{Chuyên Đề}}$ Bất đẳng thức - Cực trị

Đã gửi bởi Khoa Linh on 08-02-2018 - 21:58 trong Bất đẳng thức và cực trị

Cho x,y>0 , x+y = 3 . Tìm GTNN của P = $2x^{2}+y^{2}+\frac{28}{x}+\frac{1}{y}$

Bài này chọn điểm rơi x=2, y=1 là ra thôi 

$\left (2x^2+8 \right )+(y^2+1)+\frac{28}{x}+\frac{1}{y}-9 \geq 8x+2y+\frac{28}{x}+\frac{1}{y}-9=(7x+\frac{28}{x})+(y+\frac{1}{y})+(x+y)-9\geq 28+2+3-9=24$




#709210 $\boxed{\text{Chuyên Đề}}$ Bất đẳng thức - Cực trị

Đã gửi bởi Khoa Linh on 24-05-2018 - 20:33 trong Bất đẳng thức và cực trị

 

Cho 3 số a,b,c thỏa mãn:0 ≤ c ≤ b ≤ a ≤ 2 và a+b+c=3.

Tìm GTLN của a3 + b3 + c3

 

Ta có: 

$3a\geq a+b+c=3\Rightarrow 1\leq a\leq 2\Leftrightarrow (a-1)(a-2)\leq 0$

Ta lại có: 

$a^3+b^3+c^3=a^3+(b+c)^3-3bc(b+c)\leq a^3+(3-a)^3=9(a^2-3a+2)+9=9(a-1)(a-2)+9\leq 9$




#700246 Phương Pháp : Đặt ẩn phụ đưa về hệ phương trình đối xứng loại $2$

Đã gửi bởi Khoa Linh on 13-01-2018 - 20:29 trong Phương trình, hệ phương trình và bất phương trình

M.n cho mình hỏi với phương trình này thì đặt kiểu gì :

$x^{2}-8x+9=\sqrt{16x-24}$

 

$\large PT\Leftrightarrow (x-3)^2-2x=2\sqrt{2(x-3)+2x}$

Đặt $\large \sqrt{2(x-3)+2x}=a;x-3=b\Rightarrow 2b+2x=a^2;2a+2x=b^2$




#707228 [TOPIC] ÔN THI BẤT ĐẲNG THỨC $\boxed{\text{THPT CHUYÊN}}$...

Đã gửi bởi Khoa Linh on 28-04-2018 - 22:50 trong Tài liệu - Đề thi

105. Cho $a\,,b\,\,> 0$. Chứng minh rằng:

 

$\frac{1}{2 + a + b} + \frac{a}{2a + b + 1} + \frac{b}{2b + a + 1} \,\, \leqq \frac{3}{4}$

Anh viet9a14124869 lời giải bài này đâu cần phức tạp quá vậy ạ... 

Ta có:

$\frac{4}{a+b+2}+\frac{4a}{2a+b+1}+\frac{4b}{2b+a+1}=\frac{4}{(a+1)+(b+1)}+\frac{4a}{(a+b)+(a+1)}+\frac{4b}{(a+b)+b+1}\leq \frac{1}{a+1}+\frac{1}{b+1}+\frac{a}{a+b}+\frac{a}{a+1}+\frac{b}{a+b}+\frac{b}{b+1}=3$




#707149 [TOPIC] ÔN THI BẤT ĐẲNG THỨC $\boxed{\text{THPT CHUYÊN}}$...

Đã gửi bởi Khoa Linh on 27-04-2018 - 23:53 trong Tài liệu - Đề thi

Bài 105: Cho $a,b,c$ thỏa mãn $a\geq 4;b\geq 5,7\geq c\geq 6$ và $a^{2}+b^{2}+c^{2}=90$. Tìm giá trị nhỏ nhất $a+b+c$

 

P/s: Có 2 bài 103, nhờ DOTOANNANG sửa lại số thứ tự bài

Bài này chỉ cần điều kiện $c\geq 6$ thôi bởi vì từ giả thiết 

Ta có: 

$a\geq 4;b\geq 5\Rightarrow c\leq 7\Rightarrow (c-6)(c-7)\leq 0\Leftrightarrow 13c\geq c^2+42$

$b\geq 5;c\geq 6\Rightarrow a<9\Rightarrow (a-4)(a-9)\leq 0\Leftrightarrow 13a\geq a^2+36$

$a\geq 4;c\geq 6\Rightarrow b<8\Rightarrow (b-5)(b-8)\leq 0\Leftrightarrow 13b\geq b^2+40$

Suy ra $13(a+b+c)\geq (a^2+b^2+c^2)+42+36+40\Leftrightarrow a+b+c\geq 16$




#706959 [TOPIC] ÔN THI BẤT ĐẲNG THỨC $\boxed{\text{THPT CHUYÊN}}$...

Đã gửi bởi Khoa Linh on 25-04-2018 - 22:22 trong Tài liệu - Đề thi

 

Bài 93:

Cho 3 số thực dương a, b, c thỏa mãn $ \frac{1}{x}+\frac{1}{y}+\frac{1}{z}=2 $
Chứng minh: 
$$ \sqrt{x-1}+\sqrt{y-1}+\sqrt{z-1} \le \sqrt{x+y+z} $$ (Sưu tầm)

 

Theo Cauchy - Schwarz ta có:

$\left ( \sqrt{x-1}+\sqrt{y-1}+\sqrt{z-1} \right )^2=\left (\sum \sqrt{x}.\frac{\sqrt{x-1}}{\sqrt{x}} \right )^2$

$\leq (x+y+z)\left ( \frac{x-1}{x}+\frac{y-1}{y}+\frac{z-1}{z} \right )=(x+y+z)\left ( 3-\frac{1}{x}+\frac{1}{y}+\frac{1}{z} \right )=x+y+z$

Suy ra đpcm 




#707293 [TOPIC] ÔN THI BẤT ĐẲNG THỨC $\boxed{\text{THPT CHUYÊN}}$...

Đã gửi bởi Khoa Linh on 29-04-2018 - 15:25 trong Tài liệu - Đề thi

Bài 107:

Cho 3 số thực $a, b, c$ thỏa mãn $a+b+c=1$. Chứng minh:

$\frac{a}{{{a^2} + 1}} + \frac{b}{{{b^2} + 1}} + \frac{c}{{{c^2} + 1}} \le \frac{9}{{10}}$(Poland MO 1992)

Theo Dirichlet ta giả sử $(3b-1)(3c-1)\geq 0\Leftrightarrow b^2+c^2\leq \frac{1}{9}+\left ( b+c-\frac{1}{3} \right )^2=\frac{1}{9}+\left ( \frac{2}{3}-a \right )^2$

Ta có BĐT tương đương với: 

$\frac{a}{a^2+1}\leq \left ( \frac{1}{2}-\frac{b}{b^2+1} \right )+\left (\frac{1}{2}-\frac{c}{c^2+1} \right )-\frac{1}{10}\Leftrightarrow \frac{(b-1)^2}{b^2+1}+\frac{(c-1)^2}{c^2+1}\geq \frac{1}{5}+\frac{2a}{a^2+1}$

Mặt khác theo Cauchy - Schwarz ta có:

$\frac{(b-1)^2}{b^2+1}+\frac{(c-1)^2}{c^2+1}\geq\frac{(b+c-2)^2}{b^2+c^2+2}=\frac{(1+a)^2}{(b^2+c^2)+2}\geq \frac{(1+a)^2}{\frac{1}{9}+2+\left ( \frac{2}{3}-a \right )^2}=\frac{9(a+1)^2}{23-12a+9a^2}$

Vây ta đưa BĐT về BĐT 1 biến: 

$\frac{9(1+a)^2}{23-12a+9a^2}\geq \frac{a^2+10a+1}{5(1+a^2)}\Leftrightarrow (3a-1)^2(2a^2+2a+11)\geq 0$

Vậy ta có đpcm

p/s: Anh tr2512 xem lại hộ em nguồn bài toán bởi vì em làm 1 lần rồi và nó ghi là 1996 chứ không phải 1992. Em cảm ơn ...




#707644 [TOPIC] ÔN THI BẤT ĐẲNG THỨC $\boxed{\text{THPT CHUYÊN}}$...

Đã gửi bởi Khoa Linh on 04-05-2018 - 18:37 trong Tài liệu - Đề thi

$a+b+c+2=abc=>\frac{1}{a+1}+\frac{1}{b+1}+\frac{1}{c+1}=1$

$\left\{\begin{matrix}x=\frac{1}{a+1} \\ y=\frac{1}{b+1} \\ z=\frac{1}{c+1} \end{matrix}\right. (x,y,z> 0;x+y+z=1)$

Ta CM: $\frac{y(y+z)}{x}+\frac{z(z+x)}{y}+\frac{x(x+y)}{z}\geq 2$

Mà $\frac{y^{2}}{x}+\frac{z^{2}}{y}+\frac{x^{2}}{z}\geq x+y+z=1$

$\frac{yz}{x}+\frac{xz}{y}+\frac{xy}{z}\geq x+y+z=1$

$=>$ $\frac{y(y+z)}{x}+\frac{z(z+x)}{y}+\frac{x(x+y)}{z}\geq 2$

$<=>a=b=c=2$

Bạn Tea Coffee đã hiểu đúng ý tưởng người ra đề nhưng làm thế này sẽ nhanh và gọn hơn  :icon6:  :icon6:

Lời giải:

$\frac{a}{b+1}+\frac{b}{c+1}+\frac{c}{a+1}=\left ( \frac{a+1}{b+1}+\frac{b+1}{c+1}+\frac{c+1}{a+1} \right )-\sum \frac{1}{a+1}\geq 3-\sum \frac{1}{a+1}=2$




#710047 [TOPIC] ÔN THI BẤT ĐẲNG THỨC $\boxed{\text{THPT CHUYÊN}}$...

Đã gửi bởi Khoa Linh on 05-06-2018 - 19:40 trong Tài liệu - Đề thi

Bên đó thi chuyên chưa?

$\boxed{\text{Bài 143}}$ Cho $a,b,c >0$ ;abc=1.

Chứng minh $\frac{1}{(1+a)^2}+\frac{1}{(1+b)^2}+\frac{1}{(1+c)^2}+\frac{2}{(1+a)(1+b)(1+c)} \geq 1$

 

 

Lời giải của mình khá "vất vả" nhưng đem lại hiệu quả: 

Quy đồng ta có BĐT tương đương:

$\sum (a+1)^2(b+1)^2+2(a+1)(b+1)(c+1)\geq (a+1)^2(b+1)^2(c+1)^2$

$\Leftrightarrow (a+b+c)^2+4(a+b+c)-6abc-a^2b^2c^2-2abc(ab+bc+ca)-4abc(a+b+c)+4 \geq 0$

Thay $abc=1$ ta có BĐT trở thành: $a^2+b^2+c^2\geq 3$ (đúng theo AM-GM)

p/s: Bên mình sắp thi rồi, mùng 7,8 tháng 6. Mong bạn sớm đưa ra lời giải hay hơn  :D  :D




#708708 [TOPIC] ÔN THI BẤT ĐẲNG THỨC $\boxed{\text{THPT CHUYÊN}}$...

Đã gửi bởi Khoa Linh on 18-05-2018 - 20:16 trong Tài liệu - Đề thi

Bài 129: $a, b, c \geqslant 0, a+b+c=1$

Chứng minh: $\frac{1}{1+6a^{2}} + \frac{1}{1+6b^{2}}+\frac{1}{1+6c^{2}} \geqslant \frac{9}{5}$

 

Ta viết BĐT lại thành: $\sum \frac{25a^2}{6a^2+1}\leq 5$

Ta có: $6a^2+1=6a^2+(a+b+c)^2=2(2a^2+bc)+3a^2+b^2+c^2+2a(b+c)=2(2a^2+bc)+3a^2+b^2+c^2+2a(1-a)=2(2a^2+bc)+(a^2+b^2+c^2)+2a$

 

Áp dụng Cauchy - Schwarz ta có: 

$\sum \frac{(2a+a+2a)^2}{2(2a^2+bc)+(a^2+b^2+c^2)+2a}\leq 2\sum \frac{a^2}{2a^2+bc}+\sum \frac{a^2}{a^2+b^2+c^2}+2(a+b+c) \leq 5$

Suy ra đpcm. 




#708045 [TOPIC] ÔN THI BẤT ĐẲNG THỨC $\boxed{\text{THPT CHUYÊN}}$...

Đã gửi bởi Khoa Linh on 10-05-2018 - 20:36 trong Tài liệu - Đề thi

Bài 124: Cho a,b,c>0. Chứng minh rằng :

                                 $\frac{a^{3}}{(a+b)^{3}}+ \frac{b^{3}}{(b+c)^{3}}+\frac{c^{3}}{(a+c)^{3}}\geq \frac{3}{8}$

                                                                                     (Đề thi chọn đội tuyển quốc gia dự thi IMO 2005)

Ta viết lại BĐT dưới dạng 

$\sum \frac{1}{\left ( 1+\frac{b}{a} \right )^3}\geq \frac{3}{8}$

Đặt $\frac{b}{a}=x;\frac{c}{b}=y;\frac{a}{c}=z\Rightarrow xyz=1$

Áp dụng AM- GM ta có:

$\frac{1}{(1+x)^3}+\frac{1}{(1+x)^3}+\frac{1}{8}\geq \frac{3}{2}.\frac{1}{(x+1)^2}$

Thiết lập các BĐT tương tự ta quy về chứng minh:

$\frac{1}{(x+1)^2}+\frac{1}{(y+1)^2}+\frac{1}{(z+1)^2}\geq \frac{3}{4}$

Áp dụng BĐT phụ sau ta có:

$\frac{1}{(1+x)^2}+\frac{1}{(1+y)^2}+\frac{1}{(1+z)^2}\geq \frac{1}{xy+1}+\frac{1}{(1+z)^2}\geq \frac{3}{4}\Leftrightarrow \frac{z}{z+1}+\frac{1}{(1+z)^2}\geq \frac{3}{4}\Leftrightarrow \left ( \frac{1}{z+1}-\frac{1}{2} \right )^2\geq 0$

Suy ra ĐPCM




#706899 [TOPIC] ÔN THI BẤT ĐẲNG THỨC $\boxed{\text{THPT CHUYÊN}}$...

Đã gửi bởi Khoa Linh on 25-04-2018 - 10:40 trong Tài liệu - Đề thi

Bài 90. Cho $a\,,\,b\,,\,c$ là các số dương thỏa $a+ b+ c= 3$. Chứng minh rằng:

 

                                               1. $\prod_{cyc}^{ }(b^{2}- bc+ c^{2})\leqq 12$ (Algebraic Old & New inequalities)

Giả sử $a\geq b\geq c\geq 0$ suy ra 

$(a^2-ab+b^2)(b^2-bc+c^2)(c^2-ca+a^2)\leq (a^2-ab+b^2)b^2a^2=\frac{4}{9}.\frac{3}{2}ab.\frac{3}{2}ab.(a^2-ab+b^2)\leq \frac{4}{9}.\frac{(a+b)^6}{27}\leq \frac{4}{9}.\frac{(a+b+c)^6}{27}=12$

Dấu bằng khi a=2, b=1, c=0 và các hoán vị. (Phạm Kim Hùng)




#707600 [TOPIC] ÔN THI BẤT ĐẲNG THỨC $\boxed{\text{THPT CHUYÊN}}$...

Đã gửi bởi Khoa Linh on 03-05-2018 - 22:38 trong Tài liệu - Đề thi

Bài 119: Cho các số dương a, b, c sao cho $a+b+c+2=abc$. Chứng minh rằng:

$\frac{a}{b+1}+\frac{b}{c+1}+\frac{c}{a+1}\geq 2$

(Sưu tầm)




#706752 [TOPIC] ÔN THI BẤT ĐẲNG THỨC $\boxed{\text{THPT CHUYÊN}}$...

Đã gửi bởi Khoa Linh on 23-04-2018 - 17:29 trong Tài liệu - Đề thi

Bài 79: Cho a, b, c là các số thực dương thỏa mãn $a+b+c=3$. CMR:

$(a^2-a+1)(b^2-b+1)(c^2-c+1)\geq 1$

Cách 1(Dirichlet):

Giả sử $(b-1)(c-1)\geq 0\Rightarrow (b^2-b+1)(c^2-c+1)=bc(b-1)(c-1)+b^2+c^2-b-c+1\geq \frac{(b+c)^2}{2}-(b+c)+1=\frac{(3-a)^2}{2}-(3-a)+1=\frac{a^2-4a+5}{2}$

Vậy ta cần đi chứng minh: 

$(a^2-a+1)(a^2-4a+5)\geq 2\Leftrightarrow (a-1)^2)(a^2-3a+3)\geq 0$ luôn đúng

Cách 2(Cauchy - Schwarz):

Giả sử $c=min\left \{ a,b,c \right \}\Rightarrow c\leq 1$

Áp dụng Cauchy - Schwarz ta có:

$(a^2-a+1)(b^2-b+1)=\left ( (a-\frac{1}{2})^2+\frac{1}{2}+\frac{1}{4} \right )\left ( \frac{1}{4}+\frac{1}{2}+(b-\frac{1}{2})^2 \right )\geq\left [ \frac{1}{2}\left ( a-\frac{1}{2} \right )+\frac{1}{2}\left (b-\frac{1}{2} \right )+\frac{1}{2} \right ]^2=\frac{(3-c)^2}{4}$

Ta cần đi chứng minh: 

$(c^2-c+1)(3-c)^2\geq 4\Leftrightarrow (c-1)^2(c^2-5c+5)\geq 0$ luôn đúng do giả sử $c\leq 1$




#706465 [TOPIC] ÔN THI BẤT ĐẲNG THỨC $\boxed{\text{THPT CHUYÊN}}$...

Đã gửi bởi Khoa Linh on 19-04-2018 - 22:40 trong Tài liệu - Đề thi

Bài 49(Phạm Kim Hùng): Cho a, b, c dương và $a^2+b^2+c^2=3$. Chứng minh rằng:

$\frac{a}{a^2+2b+3}+\frac{b}{b^2+2c+3}+\frac{c}{c^2+2a+3}\leq \frac{1}{2}$

Bài 50(Phạm Kim Hùng): Cho 4 số dương a, b, c, d có tích bằng 1. Chứng minh:

$(a^2+1)(b^2+1)(c^2+1)(d^2+1)\geq (a+b+c+d)^2$




#706402 [TOPIC] ÔN THI BẤT ĐẲNG THỨC $\boxed{\text{THPT CHUYÊN}}$...

Đã gửi bởi Khoa Linh on 19-04-2018 - 15:21 trong Tài liệu - Đề thi

Bài 40: Cho các số $a,b,c>0$ thoả mãn $a+b+c=3$. Chứng minh rằng $\sum\frac{1}{a^2+2b^2+3}\leq\frac{1}{2}$

Bất đẳng thức này lần trước mình đăng nhưng bị sai nên giả thiết phải là $abc=1$

Khi đó ta có:

$\sum \frac{1}{a^2+2b^2+3}=\sum \frac{1}{(a^2+b^2)+(b^2+1)+2}\leq \frac{1}{2}\sum \frac{1}{ab+b+1}=\frac{1}{2}$




#706315 [TOPIC] ÔN THI BẤT ĐẲNG THỨC $\boxed{\text{THPT CHUYÊN}}$...

Đã gửi bởi Khoa Linh on 18-04-2018 - 19:36 trong Tài liệu - Đề thi

Bài 35: Cho $\left\{\begin{matrix} x,y,z>0 & & \\ \frac{1}{x}+\frac{1}{y}+\frac{1}{z}=4 \end{matrix}\right.$ Tìm GTLN của biểu thức:

                       P= $\frac{1}{\alpha x+\beta y+\gamma z}+\frac{1}{\beta x+\gamma y+\alpha z}+\frac{1}{\gamma x+\alpha y+\beta z}$ với $\alpha ,\beta ,\gamma \in N^{*}$.

alpha, beta, gamma lằng nhằng quá :)) mình thay bằng a,b,c. Ta có:

$\frac{1}{ ax+b y+c z}=\frac{\frac{1}{a}+\frac{1}{b}+\frac{1}{c}}{(ax+by+cz)\left ( \frac{1}{a}+\frac{1}{b}+\frac{1}{c} \right )}\leq \left ( \frac{1}{a}+\frac{1}{b}+\frac{1}{c} \right )\left ( \frac{1}{\sqrt{x}+\sqrt{y}+\sqrt{z}} \right )^2$

Tương tự thì 

$P\leq 3\left ( \frac{1}{a}+\frac{1}{b}+\frac{1}{c} \right )\left ( \frac{1}{\sqrt{x}+\sqrt{y}+\sqrt{z}} \right )^2\leq \frac{3}{81}\left( \frac{1}{a}+\frac{1}{b}+\frac{1}{c} \right )\left ( \frac{1}{\sqrt{x}}+\frac{1}{\sqrt{y}}+\frac{1}{\sqrt{z}} \right )^2$

Mặt khác:

$\sum \frac{1}{\sqrt{x}}=\sum \frac{\sqrt{\frac{1}{x}.\frac{4}{3}}}{\sqrt{\frac{4}{3}}}\leq \frac{\sum\frac{1}{x}+4 }{2\sqrt{\frac{4}{3}}}=2\sqrt{3}$

Suy ra P$\leq\frac{4}{9}\left (\frac{1}{a}+\frac{1}{b}+\frac{1}{c} \right )$

p/s: Bạn xem lại điểm rơi nhé sợ mình làm sai nhưng ý tưởng là vậy 




#706048 [TOPIC] ÔN THI BẤT ĐẲNG THỨC $\boxed{\text{THPT CHUYÊN}}$...

Đã gửi bởi Khoa Linh on 16-04-2018 - 19:14 trong Tài liệu - Đề thi

Bài 6: Cho a, b, c>0 thỏa mãn $abc=1$ Chứng minh rằng:

$a+b+c\geq \frac{1+a}{1+b}+\frac{1+b}{1+c}+\frac{1+c}{1+a}$ (Phạm Kim Hùng)

Bài 7: Cho a, b là hai số không âm thỏa mãn $a^3+b^3=2$. Chứng minh:

$3(a^4+b^4)+2a^4b^4\leq 8$ (Vasile Cirtoaje)




#706466 [TOPIC] ÔN THI BẤT ĐẲNG THỨC $\boxed{\text{THPT CHUYÊN}}$...

Đã gửi bởi Khoa Linh on 19-04-2018 - 22:50 trong Tài liệu - Đề thi

$\boxed{\text{Bài 46}}$ : Với $a,b,c>0$ min $(ab,bc,ca)\geq 1$ Chứng minh rằng

                               $(1+a^2)(1+b^2)(1+c^2)\leq (1+(\frac{a+b}{2})^2)(1+(\frac{b+c}{2})^2)(1+(\frac{c+a}{2})^2)$

Ta đi chứng minh từng BĐT rồi ghép đối xứng:

$(1+a^2)(1+b^2)\leq \left ( 1+\left ( \frac{a+b}{2} \right )^2 \right )^2\Leftrightarrow a^2+b^2+a^2b^2\leq \frac{a^2+b^2}{2}+ab+\frac{(a+b)^4}{16}$

$\Leftrightarrow \frac{(a+b)^4}{16}-(ab)^2-\frac{1}{2}(a^2+b^2-2ab)\geq 0\Leftrightarrow \left ( \frac{(a+b)^2}{4}-ab \right )\left ( \frac{(a+b)^2}{4}+ab \right )-\frac{1}{2}(a-b)^2\geq 0$

$\Leftrightarrow (a-b)^2\left ( \frac{(a+b)^2}{16}+\frac{ab}{4}-\frac{1}{2} \right )\geq 0$

Bất đẳng thức cuối cùng đúng do $ab\geq 1$

Tương tự rồi suy ra đpcm 




#706528 [TOPIC] ÔN THI BẤT ĐẲNG THỨC $\boxed{\text{THPT CHUYÊN}}$...

Đã gửi bởi Khoa Linh on 20-04-2018 - 19:55 trong Tài liệu - Đề thi

Bài 58

Cho các số thực không âm a, b, c thỏa mãn $a+b+c=3$. Chứng minh:

$\frac{a}{{\sqrt {2a + b} }} + \frac{b}{{\sqrt {2b + c} }} + \frac{c}{{\sqrt {2c + a} }} \le \sqrt 3$

Ta có BĐT tương đương: $\sum \frac{a}{\sqrt{3(2a+b)}}\leq 1$

Mà ta có: $\frac{a}{\sqrt {3(2a + b)}}=\sqrt{\frac{a^2}{3(2a+b)}}\leq \frac{1}{2}\left (\frac{a}{2a+b}+\frac{a}{3} \right )$

Nên BĐT quy về chứng minh: $\sum \frac{a}{2a+b}\leq 1\Leftrightarrow \sum \frac{b}{2a+b}\geq 1$ 

Bất đẳng thức trên đúng theo Cauchy - Schwarz: $\sum \frac{b}{2a+b}=\sum \frac{b^2}{2ab+b^2}\geq \frac{(a+b+c)^2}{(a+b+c)^2}=1$




#706707 [TOPIC] ÔN THI BẤT ĐẲNG THỨC $\boxed{\text{THPT CHUYÊN}}$...

Đã gửi bởi Khoa Linh on 22-04-2018 - 23:04 trong Tài liệu - Đề thi

Bài 77(APMO 2004): Cho a, b, c là các số thực dương. CMR: $(a^2+2)(b^2+2)(c^2+2)\geq 9(ab+bc+ca)$

Ta đi chứng minh kết quả mạnh hơn là $(a^2+2)(b^2+2)(c^2+2)\geq 3(a+b+c)^2$

Áp dụng Cauchy - Schwarz ta có:

$3(a+b+c)^2=3\left ( \frac{a+b}{\sqrt{2}}.\sqrt{2}+1.c \right )^2\leq 3\left ( \frac{(a+b)^2}{2}+1 \right )(c^2+2)$

Ta cần đi chứng minh: $(a^2+2)(b^2+2)\geq 3\left (\frac{(a+b)^2}{2}+1 \right )\Leftrightarrow \frac{(a-b)^2}{2}+(ab-1)^2\geq 0$ đúng 

Vậy hoàn tất chứng minh 




#706612 [TOPIC] ÔN THI BẤT ĐẲNG THỨC $\boxed{\text{THPT CHUYÊN}}$...

Đã gửi bởi Khoa Linh on 21-04-2018 - 21:37 trong Tài liệu - Đề thi

Bài 66: Cho các số dương a, b, c thỏa mãn $abc\leq 1$. Chứng minh rằng:

$\frac{a}{b}+\frac{b}{c}+\frac{c}{a}\geq a+b+c$

p/s: Bài không khó nhưng đẹp  :D




#706566 [TOPIC] ÔN THI BẤT ĐẲNG THỨC $\boxed{\text{THPT CHUYÊN}}$...

Đã gửi bởi Khoa Linh on 21-04-2018 - 00:27 trong Tài liệu - Đề thi

Bài 61: Cho các số thực x, y, z thỏa mãn $x^2+y^2+z^2=2$.

Chứng minh rằng: $x+y+z\leq xyz+2$

(IMO  Shortlist 1987)




#706565 [TOPIC] ÔN THI BẤT ĐẲNG THỨC $\boxed{\text{THPT CHUYÊN}}$...

Đã gửi bởi Khoa Linh on 21-04-2018 - 00:23 trong Tài liệu - Đề thi

Bài 49(Phạm Kim Hùng): Cho a, b, c dương và $a^2+b^2+c^2=3$. Chứng minh rằng:

$\frac{a}{a^2+2b+3}+\frac{b}{b^2+2c+3}+\frac{c}{c^2+2a+3}\leq \frac{1}{2}$

Bài 50(Phạm Kim Hùng): Cho 4 số dương a, b, c, d có tích bằng 1. Chứng minh:

$(a^2+1)(b^2+1)(c^2+1)(d^2+1)\geq (a+b+c+d)^2$

Mình xin đưa ra lời giải hai bài này.

Bài 49:

$\sum \frac{a}{a^2+2b+3}\leq \frac{1}{2}\sum \frac{a}{a+b+1}$

Bất đẳng thức tương tương: $\sum \frac{a}{a+b+1}\leq 1\Leftrightarrow \sum \frac{b+1}{a+b+1}\geq 2$

Áp dụng Cauchy - Schwarz ta có:

$\sum \frac{b+1}{a+b+1}\geq \frac{(a+b+c+3)^2}{(b+1)(a+b+1)+(c+1)(b+c+1)+(a+1)(c+a+1)}=1$ (khai triển tất cả kết hợp giả thiết $a^2+b^2+c^2=3$)

Bài 50:

Theo nguyên lý Dirichlet thì trong 4 số $a-1;b-1;c-1;d-1$ thì có hai số mà tích của chúng không âm. Giả sử $(b-1)(c-1)\geq 0$

Áp dụng Cauchy - Schwarz ta có:

$(a^2+1)(b^2+1)(c^2+1)(d^2+1)=(a^2c^2+a^2+c^2+1)(b^2d^2+1+b^2+d^2)\geq (abcd+a+bc+d)^2=(1+a+d+bc)^2$

Bất đẳng thức quy về chứng minh: $1+a+d+bc\geq a+b+c+d\Leftrightarrow (b-1)(c-1)\geq 0$ (đúng)




#706753 [TOPIC] ÔN THI BẤT ĐẲNG THỨC $\boxed{\text{THPT CHUYÊN}}$...

Đã gửi bởi Khoa Linh on 23-04-2018 - 17:40 trong Tài liệu - Đề thi

Bài 82: Cho 3 số dương a, b, c thỏa mãn $a+b+c=3$. Chứng minh rằng: 

$\frac{a}{ab+1}+\frac{b}{bc+1}+\frac{c}{ca+1}\geq \frac{3}{2}$

(Sưu tầm)